Jump to content

phantomladyvskaitokid's Content

There have been 184 items by phantomladyvskaitokid (Search limited from 24-05-2020)



Sort by                Order  

#313151 $\sqrt{x-1}+\sqrt{x^{2}-1}=x\sqrt{x}$

Posted by phantomladyvskaitokid on 28-04-2012 - 16:42 in Phương trình, hệ phương trình và bất phương trình

Giải các PT sau:
3) $\sqrt[3]{x+1}+\sqrt[3]{3x+1}=\sqrt[3]{x-1}$
4) $\sqrt{2x+1}+\sqrt{3-2x}=\frac{\left ( 2x-1 \right )^{2}}{2}$
5) $\sqrt{1-2x}+\sqrt{1+2x}=2-x^{2}$


3.
đặt $\sqrt[3]{x-1}=a; \sqrt[3]{x+1}=b\Rightarrow a+\sqrt[3]{2a^3+b^3}=b$

$\Leftrightarrow (a-b)^3=2a^3+b^3$

$\Leftrightarrow 3a(a^2-ab+b^2)=0$

4.

đk:...

$\sqrt{1+2x}+\sqrt{3-2x}\geq \sqrt{1+2x+3-2x}=2\Rightarrow (2x-1)^2\geq 4\Rightarrow x\geq \frac{3}{2}hoac x\leq \frac{-1}{2}$

kết hợp đk => no

5.

đk:...

$\Rightarrow 2+2\sqrt{1-4x^2}=(2-x^2)^2$

$\Rightarrow (\sqrt{1-4x^2}-1)^2+x^4=0$

$\Rightarrow x=0$



#313042 Tìm tập hợp điểm $O$

Posted by phantomladyvskaitokid on 27-04-2012 - 22:14 in Hình học

k mất tính tổng quát gs O nằm trong tg BCD

qua O kẻ đg thg //BD cắt BC, CD tại E, F

gọi I là giao điểm của AC và EF

dùng diện tích 2 tứ giác OBCD, OBAD = nhau dễ dàng c/m đc I là trg điểm của AC

suy ra O thuộc đg thg //BD và đi qua trung điểm I của AC



#313033 $$\dfrac{ab}{a+b}+\dfrac{cd}{c+d}+\dfrac{ef}{e+f}...

Posted by phantomladyvskaitokid on 27-04-2012 - 21:54 in Bất đẳng thức và cực trị

Posted Image

Posted Image



#313028 Cho $a,b,c \in (0;1]$ . Chứng minh rằng : $$\d...

Posted by phantomladyvskaitokid on 27-04-2012 - 21:23 in Bất đẳng thức và cực trị

Bài toán 2.
Cho $a,b,c$ là các số thực dương. Chứng minh rằng :
$$\left (a^3+b^3+c^3\right )\left (\dfrac{1}{a^3}+\dfrac{1}{b^3}+\dfrac{1}{c^3}\right ) \ge \dfrac{3}{2}\left (\dfrac{a+b}{c}+\dfrac{b+c}{a}+\dfrac{c+a}{b}\right )$$


$a^3+b^3+c^3\geq \frac{1}{2}\left [ ab(a+b)+bc(b+c)+ca(c+a)\right ]$

$\frac{1}{a^3}+\frac{1}{b^3}+\frac{1}{c^3}\geq \frac{3}{abc}$

$\Rightarrow$ đpcm



#313018 Tìm nghiệm nguyên dương của phương trình sau: \[8{x^3} = {3^y} + 997...

Posted by phantomladyvskaitokid on 27-04-2012 - 20:42 in Đại số

1.a,

$3^y+997\equiv 1(mod 3)\Rightarrow x^3\equiv 2(mod3)\Rightarrow x=3k+2$

$\Rightarrow 8(3k+2)^3=3^y+997\Rightarrow 9(24k^3+48k^2+32k)=3^y+933$

$\Rightarrow y=1\Rightarrow x=5$




#313017 Tìm nghiệm nguyên dương của phương trình sau: \[8{x^3} = {3^y} + 997...

Posted by phantomladyvskaitokid on 27-04-2012 - 20:41 in Đại số

1.b,

$y^2-12^2=2^x+3^2\Rightarrow 2^x\equiv y^2(mod3)\Rightarrow 2^x\equiv 1(mod3)\Rightarrow x=2k(k\in Z^{+})$

$\Rightarrow y^2-2^{2k}=153\Rightarrow (y-2^k)(y+2^k)=153$

pt ước số



#313000 Có hay ko ba số $a,b,c$ thoã mãn: $$\frac{a}{b^2-ca...

Posted by phantomladyvskaitokid on 27-04-2012 - 19:39 in Đại số

Có hay ko ba số $a,b,c$ thoã mãn:
$$\frac{a}{b^2-ca}=\frac{b}{c^2-ba}=\frac{c}{a^2-bc}$$



GS tồn tại a, b, c t/m bài toán

đk:..................

nếu trong 3 số a, b, c có 1 số =0 => a=b=c=0 (k t/m đk)

$abc\neq 0:$

$\frac{a}{b^2-ca}=\frac{b}{c^2-ba}=\frac{c}{a^2-bc}=k(k\neq 0)$

$\Rightarrow a^2+b^2+c^2=\left [ a(b^2-ca)+b(c^2-ab)+c(a^2-bc) \right ]k=0\Rightarrow a=b=c=0$ (k t/m đk)



#312849 C/M $AO\perp BI$

Posted by phantomladyvskaitokid on 26-04-2012 - 20:23 in Hình học

gọi K là trung điểm của AI $\Rightarrow \left\{\begin{matrix} KO//AH & & \\ KO=\frac{AH}{2} & & \end{matrix}\right.$

$\frac{IH}{HB}=\frac{IH}{HC}=\frac{AI}{AH}=\frac{AK}{KO}$

mà $\widehat{AKO}=\widehat{IHB}$(góc tạo bởi 2 cặp cạnh tương ứng vuông góc)

$\Rightarrow \Delta AKO\sim \Delta IHB(cgc)\Rightarrow \widehat{OAK}=\widehat{BIH}$



#312721 $MinA = \frac{x}{y} + \sqrt {1 + \frac{y}{z}} + \sq...

Posted by phantomladyvskaitokid on 25-04-2012 - 22:37 in Bất đẳng thức và cực trị

Bài 1: Cho $a,b,c,d \ge 0$ thỏa mãn:
$\left\{ \begin{array}{l}
{a^2} + 2{b^2} + 3{c^2} + 4{d^2} = 36\\
2{a^2} + {b^2} - 2{d^2} = 6
\end{array} \right.$
Tìm $MinP = {a^2} + {b^2} + {c^2} + {d^2}$



1. $\left\{ \begin{array}{l} {a^2} + 2{b^2} + 3{c^2} + 4{d^2} = 36\\ 2{a^2} + {b^2} - 2{d^2} = 6 \end{array} \right. \Rightarrow 3(a^2+b^2+c^2+d^2)=42+d^2\geq 42$

$P_{Min}=14\Leftrightarrow a=1, b=2,c=3, d=0$



#312683 Giải pt: $$2x^2-5x+2=4\sqrt{2(x^3-21x-20)}$$

Posted by phantomladyvskaitokid on 25-04-2012 - 21:03 in Phương trình, hệ phương trình và bất phương trình

Giải pt:
$$2x^2-5x+2=4\sqrt{2(x^3-21x-20)}$$


$2x^2-5x+2=4\sqrt{2(x^3-21x-20)}$

$\Leftrightarrow (2x^2-8x-10)+3(x+4)=4\sqrt{(2x^2-8x-10)(x+4)}$

đến đây đặt ẩn phụ là xong



#312659 cm các tứ giác AEHF và BCEF nội tiếp

Posted by phantomladyvskaitokid on 25-04-2012 - 19:36 in Hình học

cho tam giác ABC (AB<AC) có 3 góc nhọn nội tiếp đường tròn (O,R), 3 đường cao AD, BE, CF cắt nhau tại H.
a/ cm các tứ giác AEHF và BCEF nội tiếp (đã biết)
b/ gọi M là trung điểm của BC, K đối xứng với H qua M. cm AK là đường kính (O,R) (đã biết)
c/ EF cắt BC tại P. cm: PB.PC=PD.PM
d/ BÂC = 60 độ. Tính diện tích tam giác MEF theo R




c, tg FEMD nội tiếp nên $PD.PM=PE.PF=PB.PC$

d, $\widehat{BAC}=60^o\Rightarrow BC=\sqrt{3}R$

$\frac{EF}{BC}=\frac{AE}{AB}=\frac{1}{2}$

do đó $ME=MF=EF=\frac{BC}{2}=\frac{\sqrt{3}}{2}R\Rightarrow S_{\Delta MEF}=\frac{\sqrt{3}}{4}EF^2= \frac{3\sqrt{3}}{16}R^2$



#312654 Topic bất đẳng thức THCS (2)

Posted by phantomladyvskaitokid on 25-04-2012 - 19:20 in Bất đẳng thức và cực trị

Bài 340:
Cho $a,b,c,d$ là các số thực dương. Chứng minh rằng:
$$4abc\left [ \frac{1}{(a+b)^2c}+\frac{1}{(a+c)^2b}+\frac{1}{(c+b)^2a} \right ]+\frac{a+c}{b}+\frac{a+b}{c}+\frac{b+c}{a}\ge 9$$


$4abc\left [ \frac{1}{(a+b)^2c}+\frac{1}{(a+c)^2b}+\frac{1}{(c+b)^2a} \right ]+\frac{a+c}{b}+\frac{a+b}{c}+\frac{b+c}{a}$

$=\frac{4ab}{(a+b)^2}+\frac{4bc}{(b+c)^2}+\frac{4ca}{(c+a)^2}+\frac{a+b}{2c}+\frac{a+b}{2c}+\frac{b+c}{2a}+\frac{b+c}{2a}+\frac{c+a}{2b}+\frac{c+a}{2b}$

$\geq 9$



#312652 Chứng minh rằng: $a+b^{2}+c^{3}-ab-bc-ca\leq 1$

Posted by phantomladyvskaitokid on 25-04-2012 - 19:15 in Bất đẳng thức và cực trị

Cho các số a, b, c$\in \left [ \right 0;1]$
Chứng minh rằng: $a+b^{2}+c^{3}-ab-bc-ca\leq 1$


$a+b^2+c^3-ab-bc-ca\leq a+b+c-ab-bc-ca=(a-1)(b-1)(c-1)-abc+1\leq 1$



#312293 CM: BC, ON, AP đồng quy

Posted by phantomladyvskaitokid on 23-04-2012 - 20:09 in Hình học

Cho tam giác ABC (AB<AC) có 3 góc nhọn nội tiếp (O), các tiếp tuyến với (O) tại B và C cắt nhau tại N.
a) chứng minh: NBOC nội tiếp và NO vuông góc với BC tại I
b) Vẽ dây AM song song với BC. Đường thẳng MN cắt (O) tại điểm thứ hai là P
chứng minh: $NC^{2}=NP.NM$
c) Chứng minh: P thuộc đường tròn ngoại tiếp tam giác OMI và $\frac{BP}{AC}=\frac{CP}{AB}$
d) chứng minh: BC, ON, AP đồng quy.

- anh, em giải giúp mình câuc, d.


c, $NP.NM=NC^2=NI.NO \Rightarrow IOMP nt$

$\frac{AC}{AB}=\frac{BM}{MC}=\frac{BP}{PC}$

d, $\widehat{AIO}=\widehat{OIM}=\widehat{OPM}=\widehat{OMP}=180^o-\widehat{OIP}\Rightarrow \overline{A,I,P}$



#311921 Topic bất đẳng thức THCS (2)

Posted by phantomladyvskaitokid on 21-04-2012 - 23:33 in Bất đẳng thức và cực trị

Bài 340
Cho $a,b,c$ là độ dài 3cạnh tam giác thoã mãn $a+b+c=1$.CMR:
$$1<\frac{b}{\sqrt{a+b^2}}+\frac{c}{\sqrt{b+c^2}}+\frac{a}{\sqrt{c+a^2}}<2$$


$a+b^2=a(a+b+c)+b^2<a(a+b+c)+b^2+c^2+ab+ac+2bc=(a+b+c)^2=1 \Rightarrow \frac{b}{\sqrt{a+b^2}}>b$

tuong tu suy ra $1<\frac{b}{\sqrt{a+b^2}}+\frac{c}{\sqrt{b+c^2}}+\frac{a}{\sqrt{c+a^2}}$



#311919 tìm quỹ tích của điểm N khi M chạy trên d

Posted by phantomladyvskaitokid on 21-04-2012 - 23:20 in Hình học

$OA.OB=R^2\Rightarrow$B cố định $\Rightarrow$ N thuộc đường tròn đường kính OB



#311887 Topic bất đẳng thức THCS (2)

Posted by phantomladyvskaitokid on 21-04-2012 - 21:35 in Bất đẳng thức và cực trị

Bài 339:
Tìm giá trị lớn nhất và nhỏ nhất của biểu thức:
$$\frac{x-y}{x^4+y^4+6}$$


$\left | \frac{x-y}{x^4+y^4+6} \right |\leq \frac{|x-y|}{2(x^2+y^2)+4} \leq \frac{|x-y|}{(x-y)^2+4}\leq 4\Rightarrow -4\leq \frac{x-y}{x^4+y^4+6}\leq 4$



#311823 Trận 10 - "MSS14 daovuquang" VS ALL

Posted by phantomladyvskaitokid on 21-04-2012 - 17:50 in Thi giải toán Marathon cấp THCS 2012

Dựng hình bình hành BICA'

Gọi M, N thứ tự là trung điểm của BC, B'C'; G là giao của A'N và AM

$\vartriangle BIC=\vartriangle CA'B\Rightarrow \vartriangle BIC \sim \vartriangle AB'C\Rightarrow \left\{\begin{matrix} \widehat{BCI}=\widehat{ACB'} & & \\ \frac{IC}{BC} =\frac{B'C}{AC}& & \end{matrix}\right.\Rightarrow \vartriangle IB'C\sim \vartriangle BAC(c.g.c)$

do đó $\frac{IB'}{B'C}=\frac{AB}{AC}=\frac{AC'}{B'C}\Rightarrow IB'=AC'$

c/m tương tự ta đc $AB'=C'I$

suy ra AC'IB' là hình bình hành $\Rightarrow$ N là trung điểm của AI

$\vartriangle AIA'$ có trung tuyến AM và A'N cắt nhau tại G nên G là trọng tâm $\vartriangle AIA'$

$\Rightarrow \frac{AG}{AM}=\frac{A'G}{A'N}=\frac{2}{3}$

do đó G là trọng tâm của tg ABC và A'B'C' (đpcm)

Mở rộng

Gọi D, E, F lần lượt là chân đg vg góc hạ từ A', B', C' xuống BC, CA, AB thì G cũng là trọng tâm tg DEF

p/s: k thấy kí hiệu đồng dạng nên dùng ~

Một số lỗi lý luận nhỏ và trình bày. Sao không thấy lời chứng minh cho mở rộng?
D-B=8.8h
E=10
F=0
S=69.2

Attached Images

  • h5.png



#311786 Sự cố khi thi Violympic

Posted by phantomladyvskaitokid on 21-04-2012 - 12:58 in Cuộc thi VIOlympic (Cuộc thi do Bộ giáo dục và đào tạo tổ chức)

đề giống nhau chứ



#311562 Tìm Min của t biết x,y,z,t $ \in R$ thoả$x + y + z + t =...

Posted by phantomladyvskaitokid on 19-04-2012 - 20:47 in Bất đẳng thức và cực trị

Cho $x ; y ; z ; t$ là các số thực đồng thời thỏa mãn các đk sau :
$x + y + z + t = 8 ; xy + xz + xt + yz + yt + zt = 18$
Tìm giá trị nhỏ nhất mà t có thể đạt được.


$x^2+y^2+z^2+t^2=(x+y+z+t)^2-2(xy+xz+xt+yz+yt+tz)=28$

$\Rightarrow 3(28-t^2)=3(x^2+y^2+z^2)\geq (x+y+z)^2=(8-t)^2$

$\Rightarrow(t+1)(t-5)\leq 0\Rightarrow -1 \leq t \leq 5$



#311551 $abc=1$. Chứng minh rằng : $$5+\dfrac{a}{b}+\d...

Posted by phantomladyvskaitokid on 19-04-2012 - 20:29 in Bất đẳng thức và cực trị

Ví dụ 3:VMO 2002-Trần Nam Dũng
Chứng minh rằng với mọi $a,b,c\geq 0$,ta có:
$2(a^2+b^2+c^2)+abc+8\geq 5(a+b+c)$
Lời giải:
Sử dụng bất đẳng thức AM-GM,ta có:
$12(a^2+b^2+c^2)+6abc+48-30(a+b+c)$
$=12(a^2+b^2+c^2)+3(2abc+1)+45-5.2.3(a+b+c)$
$\geq12(a^2+b^2+c^2)+9\sqrt[3]{a^2b^2c^2}+45-5.((a+b+c)^2+9)$
$=7(a^2+b^2+c^2)+\frac{9abc}{\sqrt[3]{abc}}-10(ab+bc+ca)$
$\geq 7(a^2+b^2+c^2)+\dfrac{27abc}{a+b+c}-10(ab+bc+ca)
$
Mặt khác sử dụng bất đẳng thức Schur,
$\dfrac{9abc}{a+b+c}\geq 4(ab+bc+ca)-(a+b+c)^2=2(ab+bc+ca)-(a^2+b^2+c^2)$
Do đó
$7(a^2+b^2+c^2)+\dfrac{27abc}{a+b+c}-10(ab+bc+ca)$
$ \geq 7(a^2+b^2+c^2)+6(ab+bc+ca)-3(a^2+b^2+c^2)-10(ab+bc+ca)=4(a^2+b^2+c^2-ab-bc-ca)\geq 0$
Bất đẳng thức được chứng minh.




#311099 Cho $a,b,c > 0 $ thỏa mãn : $a+b+c =3$. CMR :

Posted by phantomladyvskaitokid on 17-04-2012 - 18:54 in Bất đẳng thức - Cực trị



ta co $(a+b)(b+c)(c+a)=(a+b+c)(ab+bc+ca)-abc=3(ab+bc+ca)-abc$

$\Rightarrow \frac{3(a+b)(b+c)(c+a)}{8(ab+bc+ca)}=\frac{9}{8}-\frac{3abc}{8(ab+bc+ca)}$

ma $\frac{3abc}{8(ab+bc+ca)}\leq \frac{3abc}{24\sqrt[3]{a^2b^2c^2}}=\frac{\sqrt[3]{abc}}{8}\leq \frac{1}{8}$

do do $\sqrt[2012]{\frac{3(a+b)(b+c)(c+a)}{8(ab+bc+ca)}}\geq 1\geq \sqrt[2011]{\frac{ab+bc+ca}{a^2+b^2+c^2}}$



#311024 $abc=1$. Chứng minh rằng : $$\dfrac{a}{b+c+1}+...

Posted by phantomladyvskaitokid on 17-04-2012 - 10:34 in Bất đẳng thức và cực trị

Bài toán 1.
Cho $a, b, c > 0, abc=1$. Chứng minh rằng :
$$\dfrac{a}{b+c+1}+\dfrac{b}{c+a+1}+\dfrac{c}{a+b+1}\ge 1$$
Bài toán 2.
Cho $a,b,c$ là các số thực dương. Chứng minh rằng :
$$\dfrac{a+b}{c}+\dfrac{c+a}{b}+\dfrac{a+b}{c}\ge 4\left (\dfrac{a}{b+c}+\dfrac{b}{a+c}+\dfrac{c}{a+b}\right )$$


1. $3(a^2+b^2+c^2)\geq (a+b+c)^2\geq 3(a+b+c\Rightarrow a+b+c\leq a^2+b^2+c^2$

$\frac{a}{b+c+1}+\frac{b}{c+a+1}+\frac{c}{a+b+1}\geq \frac{(a+b+c)^2}{2(ab+bc+ca)+a+b+c}\geq \frac{(a+b+c)^2}{(a+b+c)^2}=1$

2. $\frac{a+b}{c}+\frac{b+c}{a}+\frac{c+a}{b}=a(\frac{1}{b}+\frac{1}{c})+b(\frac{1}{c}+\frac{1}{a})+c(\frac{1}{a}+\frac{1}{b})\geq 4(\frac{a}{b+c}+\frac{b}{c+a}+\frac{c}{a+b})$



#311019 Chứng minh rằng: $x(1-y)+y(1-z) +z(1-t)+t(1-x) \leq 2$

Posted by phantomladyvskaitokid on 17-04-2012 - 09:12 in Bất đẳng thức và cực trị

bđt cần c/m tđ vs $x+y+z+t-(x+z)(y+t)\leq 2$

đặt $x+z=a; y+t=b \Rightarrow a;b \in \left [ 0;2 \right ]$

$\Rightarrow (a-2)(b-2)\geq 0\Rightarrow 2a+2b-ab \leq 4$

mà $2a+2b-2ab \leq 2a+2b-ab$

do đó $a+b-ab \leq 2$

ta đc đpcm



#310949 $\frac{a}{b+c-a}+\frac{b}{a+c-b}+\frac{c}{a+b-c}\geq...

Posted by phantomladyvskaitokid on 16-04-2012 - 21:34 in Bất đẳng thức và cực trị

dat $\left\{\begin{matrix} b+c-a=x & & \\ c+a-b=y & & \\ a+b-c=z & & \end{matrix}\right.\Rightarrow \left\{\begin{matrix} a=\frac{y+z}{2} & & \\ b=\frac{z+x}{2} & & \\ c=\frac{x+y}{2} & & \end{matrix}\right.$

bdt can c/m tt $\frac{y+z}{2x}+\frac{z+x}{2y}+\frac{x+y}{2z}\geq 3$

dung theo cauchy